彈性

解決問題以獲得最優價格(最大化利潤)嘗試內部

  • December 1, 2020

讓需求函式定義為 $ D(p)=B-bp $ , 在哪裡 $ b,B>0 $ . 一家公司有一些生產成本, $ c $ ,並且可以設置價格 $ p $ 在需求的約束下。

  • 最優價格是多少?
  • 價格彈性是高於還是低於 $ 1 $ (絕對值)
  • 價格如何取決於 $ b $ ?

如何解決這樣一個最大化問題?

我的嘗試

彈性將由下式給出

$ \frac{p}{q}\cdot(-b) $ 自從 $ D’(p)=-b $

那麼問題來了 $$ \begin{equation*} max_p(p-c)D’(p) \end{equation*} $$

然後遵循

$$ \begin{equation*} D(p)+(p-c)D’(p)=0 \Leftrightarrow \end{equation*} $$

$$ \begin{equation*} D(p)\left ( 1+(p-c)\frac{\epsilon}{p} \right )=0\Leftrightarrow \end{equation*} $$

然後是最優價格 $ p^* $ $$ \begin{equation*} p^=\frac{\epsilon}{1+\epsilon}c=\frac{\left | \epsilon \right |}{\left | \epsilon \right |-1}c=\frac{\left | \frac{-bp}{q} \right |}{\left | \frac{-bp}{q} \right |-1}c=\frac{c\left | b \right |\left | p \right |}{\left | b\right |\left |p \right |-\left | q \right |} \end{equation} $$

然後我們注意到

$$ \begin{equation*} \left | \epsilon \right |>1 \end{equation*} $$

…………………………………………………………………………………..

我覺得這既不正確也不正確。幫助表示讚賞。

從建立利潤方程開始:

$$ \max_p \pi = (p-c)D(q) $$

然後將需求替換為:

$$ \pi = (p-c)(B-bp) $$

取利潤的導數並將其等同於 0:

$$ \frac{d \pi}{dp}=B-2pb + cb=0 $$

現在求解最優價格:

$$ p^*= \frac{B+cb}{2b} $$

還有另一種方法如何獲得使用需求彈性的相同解決方案。

眾所周知的壟斷最優定價規則(參見 Peitz 和 Belleflamme 工業組織:市場和策略)說最優壟斷價格由下式給出:

$$ \frac{p-mc}{p}=-\frac{1}{El} $$

在您的情況下,邊際成本 (mc) 是 $ c $ ,你的需求彈性實際上是:

$$ EL= -\frac{bp}{q}= -\frac{bp}{B-bp} $$

在您的嘗試中,您忘記在彈性中替換 Q。

將這些插入最優定價規則給我們:

$$ \frac{p-c}{p}=-\frac{1}{\left( -\frac{bp}{B-bp} \right) } $$

什麼時候解決 $ p $ 再次給我們:

$$ p^*= \frac{B+cb}{2b} $$

兩種方法都給出了相同的解決方案——在我看來,在這種情況下,第一種方法更快。

現在關於我們所說的價格和參數 $ b $ ,我們可以將最優價格公式重新排列為:

$$ p^*= \frac{B}{2b}+\frac{c}{2} $$

我們可以很清楚地看到,當 $ b $ 參數增加最優價格降低。

我們還可以通過檢查的導數更正式地證明它 $ p^* $ 關於 $ b $ 這給了我們:

$$ \frac{d p^*}{d b} = -\frac{B}{2b^2} $$

由於衍生品是負數,我們確認價格隨 $ b $ .

接下來,根據關於什麼是均衡價格的彈性的評論請求,我們可以通過將均衡價格代入我們已經在上面計算的彈性來計算:

$$ EL(p^)= -\frac{b p^ }{B-bp^*}= -\frac{b \left( \frac{B+cb}{2b} \right)}{B-b \left( \frac{B+cb}{2b} \right)} \= - \frac{B+bc}{B-bc} $$

最後一個表達式的絕對值將大於 1,因為分子將大於分母 ( $ B+cb>B-bc $ ) 和絕對值將使值變為正值。

引用自:https://economics.stackexchange.com/questions/41220